Download Chapter 27

Survey
yes no Was this document useful for you?
   Thank you for your participation!

* Your assessment is very important for improving the workof artificial intelligence, which forms the content of this project

Document related concepts

Classical mechanics wikipedia , lookup

Maxwell's equations wikipedia , lookup

Introduction to gauge theory wikipedia , lookup

Standard Model wikipedia , lookup

Force wikipedia , lookup

Lepton wikipedia , lookup

Field (physics) wikipedia , lookup

Casimir effect wikipedia , lookup

Speed of gravity wikipedia , lookup

Magnetic field wikipedia , lookup

Neutron magnetic moment wikipedia , lookup

Circular dichroism wikipedia , lookup

History of subatomic physics wikipedia , lookup

Superconductivity wikipedia , lookup

Fundamental interaction wikipedia , lookup

Newton's laws of motion wikipedia , lookup

Newton's theorem of revolving orbits wikipedia , lookup

Elementary particle wikipedia , lookup

Electric charge wikipedia , lookup

Anti-gravity wikipedia , lookup

Electromagnet wikipedia , lookup

Magnetic monopole wikipedia , lookup

Electromagnetism wikipedia , lookup

Electrostatics wikipedia , lookup

Aharonov–Bohm effect wikipedia , lookup

Classical central-force problem wikipedia , lookup

Work (physics) wikipedia , lookup

Lorentz force wikipedia , lookup

Transcript
Chapter 27
Homework Solutions
Young and Freedman, 13th Edition
27.1.IDENTIFY and SET UP: Apply Eq. (27.2) to calculate F . Use the cross products of unit vectors from
Section 1.10.
EXECUTE: v  (419 104 m/s)iˆ  (385 104 m/s) ˆj
(a) B  (140 T)iˆ
F  qv  B  (124 108 C)(140 T)[(419 104 m/s)iˆ  iˆ  (385 104 m/s) ˆj  iˆ]
iˆ  iˆ  0, ˆj  iˆ  kˆ
F  (124 108 C)(140 T)(385 104 m/s)(kˆ)  (668 104 N)kˆ
EVALUATE: The directions of v and B are shown in Figure 27.1a.
The right-hand rule gives that v  B is
directed out of the paper ( z-direction).
The charge is negative so F is opposite
to v  B.
Figure 27.1a
F is in the  z-direction. This agrees with the direction calculated with unit vectors.
(b) EXECUTE: B  (140 T)kˆ
F  qv  B  (124 108 C)(140 T)[(419 104 m/s)iˆ  kˆ  (385 104 m/s) ˆj  kˆ ]
iˆ  kˆ   ˆj, ˆj  kˆ  iˆ
F  (727 104 N)( ˆj)  (668 104 N)iˆ  [(668 104 N)iˆ  (727 104 N) ˆj]
EVALUATE: The directions of v and B are shown in Figure 27.1b.
The direction of F is opposite to v  B since
q is negative. The direction of F computed
from the right-hand rule agrees qualitatively
with the direction calculated with unit vectors.
Figure 27.1b
27.3.
IDENTIFY: The force F on the particle is in the direction of the deflection of the particle. Apply the
right-hand rule to the directions of v and B. See if your thumb is in the direction of F , or opposite to
that direction. Use F  q v B sin  with   90 to calculate F.
SET UP: The directions of v , B and F are shown in Figure 27.3.
EXECUTE: (a) When you apply the right-hand rule to v and B, your thumb points east. F is in this
direction, so the charge is positive.
(b) F  q v B sin   (850 106 C)(475 103 m/s)(125 T)sin90  00505 N
EVALUATE: If the particle had negative charge and v and B are unchanged, the particle would be
deflected toward the west.
Figure 27.3
27.11.IDENTIFY and SET UP:  B   B  dA
Circular area in the xy-plane, so A   r 2   (00650 m)2  001327 m2 and dA is in the z -direction.
Use Eq. (1.18) to calculate the scalar product.
EXECUTE: (a) B  (0230 T)kˆ; B and dA are parallel (  0) so B  dA  BdA.
B is constant over the circular area so
 B   B  dA   BdA  B  dA  BA  (0230 T)(001327 m 2 )  305  10 3 Wb
(b) The directions of B and dA are shown in Figure 27.11a.
B  dA  B cos  dA
with   531
Figure 27.11a
B and  are constant over the circular area so  B   B  dA   B cos  dA  B cos   dA  B cos  A
B  (0230 T)cos531(001327 m2 )  183 103 Wb.
(c) The directions of B and dA are shown in Figure 27.11b.
B  dA  0 since dA and B are perpendicular (  90)
 B   B  dA  0.
Figure 27.11b
EVALUATE: Magnetic flux is a measure of how many magnetic field lines pass through the surface. It
is maximum when B is perpendicular to the plane of the loop (part a) and is zero when B is parallel
to the plane of the loop (part c).
27.15.(a) IDENTIFY: Apply Eq. (27.2) to relate the magnetic force F to the directions of v and B The
electron has negative charge so F is opposite to the direction of v  B. For motion in an arc of a
circle the acceleration is toward the center of the arc so F must be in this direction. a  v2/R.
SET UP:
As the electron moves in the semicircle,
its velocity is tangent to the circular path.
The direction of v0  B at a point along
the path is shown in Figure 27.15.
Figure 27.15
EXECUTE: For circular motion the acceleration of the electron arad is directed in toward the center of
the circle. Thus the force FB exerted by the magnetic field, since it is the only force on the electron,
must be radially inward. Since q is negative, FB is opposite to the direction given by the right-hand
rule for v0  B. Thus B is directed into the page. Apply Newton’s second law to calculate the
magnitude of B:  F  ma gives  Frad  ma FB  m(v2/R)
FB  q v B sin   q v B, so q v B  m(v 2/R)
B
mv (9109  1031 kg)(141  106 m/s)

 160  104 T
qR
(1602  1019 C)(0050 m)
(b) IDENTIFY and SET UP: The speed of the electron as it moves along the path is constant. ( FB
changes the direction of v but not its magnitude.) The time is given by the distance divided by v0 .
EXECUTE: The distance along the semicircular path is  R, so
 R  (0050 m)
t

 111 107 s.
v0 141106 m/s
EVALUATE: The magnetic field required increases when v increases or R decreases and also depends
on the mass to charge ratio of the particle.
27.21.(a) IDENTIFY and SET UP: Apply Newton’s second law, with a  v 2 /R since the path of the particle is
circular.
EXECUTE:
v
q BR
m

 F  ma says q v B  m(v 2 /R)
(1602  1019 C)(250 T)(696 10 3 m)
334  1027 kg
 835  105 m/s
(b) IDENTIFY and SET UP: The speed is constant so t  distance/v.
R
 (696  103 m)
 262  108 s
v
835 105 m/s
(c) IDENTIFY and SET UP: kinetic energy gained  electric potential energy lost
EXECUTE: t 
EXECUTE:
V
1 mv 2
2

 qV
mv 2 (334  1027 kg)(835  105 m/s) 2

 727  103 V  727 kV
19
2q
2(1602  10
C)
EVALUATE: The deutron has a much larger mass to charge ratio than an electron so a much larger B
is required for the same v and R. The deutron has positive charge so gains kinetic energy when it goes
from high potential to low potential.
27.28.IDENTIFY: For no deflection the magnetic and electric forces must be equal in magnitude and opposite in
direction.
SET UP: v  E/B for no deflection. With only the magnetic force, q v B  mv 2 /R.
EXECUTE: (a) v  E/B  (156 104 V/m)/(462 103 T)  338 106 m/s.
(b) The directions of the three vectors v , E and B are sketched in Figure 27.28.
(c) R 
mv (911  1031 kg)(338  106 m/s)

 417  103 m.
qB
(160  1019 C)(462  103 T)
2 m 2 R 2 (417  103 m)


 774  109 s.
6
qB
v
(338  10 m/s)
EVALUATE: For the field directions shown in Figure 27.28, the electric force is toward the top of the
page and the magnetic force is toward the bottom of the page.
T
Figure 27.28
27.31.IDENTIFY: For the alpha particles to emerge from the plates undeflected, the magnetic force on them must
exactly cancel the electric force. The battery produces an electric field between the plates, which acts
on the alpha particles.
SET UP: First use energy conservation to find the speed of the alpha particles as they enter the region
between the plates: qV  1/2 mv2. The electric field between the plates due to the battery is E  Vbd . For
the alpha particles not to be deflected, the magnetic force must cancel the electric force, so qvB  qE,
giving B  E/v.
EXECUTE: Solve for the speed of the alpha particles just as they enter the region between the plates.
Their charge is 2e.
v 
2(2e)V
4(160  1019 C)(1750V)

 411 105 m/s
m
664  1027 kg
The electric field between the plates, produced by the battery, is
E  Vb /d  (150 V)/(0.00820 m)  18,300 V/m
The magnetic force must cancel the electric force:
B  E/v  (18,300 V/m)/(4.11105 m/s)  0.0445 T
The magnetic field is perpendicular to the electric field. If the charges are moving to the right and the
electric field points upward, the magnetic field is out of the page.
EVALUATE: The sign of the charge of the alpha particle does not enter the problem, so negative
charges of the same magnitude would also not be deflected.
27.41.IDENTIFY and SET UP: The magnetic force is given by Eq. (27.19). FI  mg when the bar is just ready to
levitate. When I becomes larger, FI  mg and FI  mg is the net force that accelerates the bar upward.
Use Newton’s second law to find the acceleration.
mg (0750 kg)(980 m/s2 )

 3267 A
lB
(0500 m)(0450 T)
 IR  (32.67 A)(25.0 )  817 V
(a) EXECUTE: IlB  mg , I 

(b) R  20,I   /R  (8167 V)/(20)  408 A
FI  IlB  92 N
a  (FI  mg )/ m  113 m/s2
EVALUATE: I increases by over an order of magnitude when R changes to FI  mg and a is an
order of magnitude larger than g.
27.48.IDENTIFY:     B and U    B cos  , where   NIB.    B sin  .
SET UP:  is the angle between B and the normal to the plane of the loop.
EXECUTE: (a)   90.   NIABsin(90)  NIAB, direction kˆ  ˆj  iˆ. U   B cos  0.
(b)   0.   NIABsin(0)  0, no direction. U   Bcos  NIAB.
(c)   90.   NIAB sin(90)  NIAB, direction  kˆ  ˆj  iˆ. U   B cos  0.
(d)   180:   NIAB sin(180)  0, no direction, U   B cos(180)  NIAB.
EVALUATE: When  is maximum, U  0. When U is maximum,   0.